2
$\begingroup$

Let $A\subset\mathbb{Z}/n\mathbb{Z}$ such that: $|A|>n^{d}$ ($0< d <1$).

Let $C=\{(x,y,2y-x)\in A\times A \times A\}$ be the set of $3$-term arithmetic progressions within $A$.

[The original version asked about $x+y \in A$, settled by the example of Anthony Quas.]

I need to prove (or refute) that there exists a lower bound $u(n)$ on $\frac{|C|}{|A|} $ such that

$$\lim_{n\rightarrow\infty}\frac{\log(u(n))}{\log(n)}>0.$$

thanks to the helpers

$\endgroup$

3 Answers 3

11
$\begingroup$

With your corrected question you are asking, in a strange way, for the number of arithmetic progressions of length 3 in A. There is a well-known example of Behrend of a set of size $n/\exp(c\sqrt{\log n})$ that contains no non-degenerate APs of length 3. So the answer to your question is no.

Edit: now that you have rephrased your question explicitly to be about arithmetic progressions of length 3, the words "in a strange way" no longer apply above. Indeed, the whole of the first sentence is rendered redundant (but I'll leave it there for the historical record).

$\endgroup$
3
$\begingroup$

If $A$ is the set of odd numbers up to $n/2$ then $C$ is empty.

$\endgroup$
1
  • $\begingroup$ $A = [\lfloor n/3 \rfloor+1,\lfloor 2n/3 \rfloor]$ also works for the original problem. $\endgroup$ Jun 20, 2011 at 2:43
0
$\begingroup$

yes, you are right of course, i will correct my question: the group D is: D={(x,y)|2x-y∈A} so now C is at lesat the size of A [(a,a) is in C for every element of A]

$\endgroup$

Your Answer

By clicking “Post Your Answer”, you agree to our terms of service and acknowledge you have read our privacy policy.

Not the answer you're looking for? Browse other questions tagged or ask your own question.